Important Part of Gradient Descent Convergence Proof

By Bolun Dai | Feb 10th 2021

This blog post proves a crucial statement in the gradient descent convergence analysis: from the assumption \(f\) is convex, twice differentiable function and \(\nabla{f}\) is Lipschitz continuous with constant \(L>0\) we have

\[f(y) \leq f(x) + \nabla{f}(x)^T(y - x) + \frac{L}{2}\|y-x\|_2^2.\]

Here we list four statements:

  • (1) \(\nabla{f}\) is Lipschitz continuous with constant \(L>0\)
  • (2) \((\nabla{f}(x)-\nabla{f}(y))^T(x-y)\leq L\|x-y\|_2^2\)
  • (3) \(\nabla^2{f}(x)\preceq LI\)
  • (4) \(f(y) \leq f(x) + \nabla{f}(x)^T(y - x) + \frac{L}{2}\|y-x\|_2^2\)

The outline of the proof is \(\mathrm{i}\rightarrow \mathrm{iii} \rightarrow \mathrm{iv}\) but the proof of \(\mathrm{i}\rightarrow \mathrm{ii}\) is also given.

i → ii

From \(\nabla{f}\) is Lipschitz continuous with constant \(L>0\) we know \(\|\nabla{f}(x) - \nabla{f}(y)\|_2\leq L\|x-y\|_2\). From the Cauchy-Schwartz inequality we have

\[|\langle u, v\rangle| \leq \|u\|_2\|v\|_2.\]

Setting \(u = \nabla{f}(x) - \nabla{f}(y)\) and \(v = x - y\) we have

\[\begin{align*} |(\nabla{f}(x) - \nabla{f}(y))^T(x - y)| &\leq \|\nabla{f}(x) - \nabla{f}(y)\|_2\|x - y\|_2\\ &\leq L\|x-y\|_2\|x - y\|_2 & \mathrm{Lipschitz\ condition}\\ &\leq L\|x-y\|_2^2. \end{align*}\]

Since we have the absolute value of \((\nabla{f}(x) - \nabla{f}(y))^T(x - y)\) less than or equal to \(L\|x-y\|_2^2\), we definetely have

\[(\nabla{f}(x)-\nabla{f}(x))^T(x-y)\leq L\|x-y\|_2^2.\]

i → iii

From the mean value theorem we can have

\(\frac{\nabla f(x + \Delta{x}) - \nabla f(x)}{\Delta{x}} = \nabla^2f(x + t\Delta{x}),\ t\in[0, 1].\) If we multiply both sides with \(\Delta{x}\) and take the norm on both sides gives us

\[\begin{align*} \|\nabla^2f(x + t\Delta{x})\Delta{x}\| &= \|\nabla f(x + \Delta{x}) - \nabla f(x)\|\\ \|\nabla^2f(x + t\Delta{x})\|\|\Delta{x}\| &\leq L\|\Delta{x}\|\\ \|\nabla^2f(x + t\Delta{x})\| &\leq L\\ \|\nabla^2f(x)\| &\leq L & \mathrm{set}\ t\rightarrow 0\ \end{align*}\]

Since \(f\) is a convex function, we know its Hessian is positive semidefinite. Thus, \(\|\nabla^2f(x)\|\) represents the spectral norm, which is equal to its largest eigenvalue. Thus we have \(\nabla^2{f}(x)\preceq LI\). See definition 7 and fact 8 of this note.

iii → iv

From the second-order Taylor series we have

\[f(y) = f(x) + \nabla f(x)^T(y - x) + \frac{1}{2}(y - x)^T\nabla^2{f}(x)(y - x).\]

From \(\nabla^2{f}(x)\preceq LI\) we have

\[\frac{1}{2}(y - x)^T\nabla^2{f}(x)(y - x) \leq \frac{1}{2}(y - x)^TLI(y - x) = \frac{L}{2}\|y-x\|_2^2.\]

Thus we have

\[f(y) \leq f(x) + \nabla{f}(x)^T(y - x) + \frac{L}{2}\|y-x\|_2^2.\]

i → iv

Prof.Joan Bruna’s mentioned in his class a way to prove iv from i which I did not think of before, thus I will include it here. If we have \(F(t) = f\big(y + t(x-y)\big)\), then using the fundamental theorem of calculus

\[F(b) - F(a) = \int_{b}^{a}{f(t)dt},\ \mathrm{if\ }F^\prime(t) = f(t),\]

we can have

\[\begin{align} F(b) - F(a) &= \int_{b}^{a}{F^\prime(t)dt}\\ &= \int_{b}^{a}{(x-y)\nabla{f}\big(y + t(x-y)\big)dt}. \end{align}\]

By setting \(a = 1\) and \(b = 0\) we have

\[F(b) - F(a) = f(x) - f(y) = \int_{0}^{1}{(x-y)\nabla{f}\big(y + t(x-y)\big)dt},\]

which can be written as

\[f(x) - f(y) = \int_{0}^{1}{\Big\langle\nabla{f}\big(y + t(x-y)\big), (x-y)\Big\rangle dt}.\]

Also we need to note that for a constant \(s\) or function \(s(x)\) with no \(t\) in the input, we have \(F(x) = s(x)t\) and \(dF(x)/dt = s(x)\). Thus, using the fundamental theorem of calculus we have

\[s(x) = \int_{0}^{1}s(x)dt = s(x)t\Big|_0^1 = s(x) - 0.\]

Therefore, we have

\[\begin{align} \Bigg|f(x) - f(y) - \langle\nabla f(y), x-y\rangle\Bigg| &= \Bigg|\int_{0}^{1}{\Big\langle\nabla{f}\big(y + t(x-y)\big), x-y\Big\rangle dt} - \int_{0}^{1}{\langle\nabla f(y), x-y\rangle dt}\Bigg|\\ &= \Bigg|\int_{0}^{1}{\Big\langle\Big[\nabla{f}\big(y + t(x-y)\big) - \nabla f(y)\Big], x-y\Big\rangle dt}\Bigg|\\ &\leq \int_{0}^{1}{\Bigg|\Big\langle\Big[\nabla{f}\big(y + t(x-y)\big) - \nabla f(y)\Big], x-y\Big\rangle\Bigg|dt}\\ &\leq \int_{0}^{1}{\Big\|\nabla{f}\big(y + t(x-y)\big) - \nabla f(y)\Big\|\Big\|x-y\Big\|dt}\\ &\leq \int_{0}^{1}{L\Big\|t(x-y)\Big\|\Big\|x-y\Big\|dt}\\ &= L\|x-y\|^2\int_{0}^{1}{tdt}\\ &= \frac{L}{2}\|x-y\|^2. \end{align}\]

Then we finally have

\[f(x) \leq f(y) + \langle\nabla f(y), x-y\rangle + \frac{L}{2}\|x-y\|^2,\ \forall x, y.\]

Credit to Ryan Tibshirani, powered by Powered by MathJax